Középiskolai Matematikai és Fizikai Lapok
Informatika rovattal
Kiadja a MATFUND Alapítvány
Már regisztráltál?
Új vendég vagy?

Fórum: GEOMETRIA

  [1]    [2]    [3]    [4]    [5]    [6]    [7]    [8]    [9]    [10]    [11]    [12]    [13]    [14]    [15]    [16]    [17]    [18]    [19]    [20]    [21]    [22]    [23]    [24]    [25]    [26]    [27]    [28]    [29]    [30]    [31]    [32]    [33]    [34]    [35]    [36]    [37]    [38]    [39]    [40]    [41]    [42]    [43]    [44]    [45]    [46]    [47]    [48]    [49]    [50]    [51]    [52]    [53]    [54]    [55]    [56]    [57]    [58]    [59]    [60]    [61]    [62]    [63]    [64]    [65]    [66]    [67]    [68]    [69]    [70]    [71]    [72]    [73]    [74]    [75]    [76]    [77]    [78]  

Szeretnél hozzászólni? Jelentkezz be.
[1825] w2014-03-03 08:06:55

Talán a leggyorsabb megoldási lehetőség azt használja ki, hogy ABCD akkor és csak akkor érintőnégyszög, ha

\tg\frac{ABD\angle}2\cdot\tg\frac{BDC\angle}2=\tg\frac{ADB\angle}2\cdot\tg\frac{DBC\angle}2.

Ennek ellenére úgy emlékszem, hogy számítások nélküli megoldás is van, de azt direkt nem olvastam el. (Monge-tétel?...)

Előzmény: [1824] Sinobi, 2014-03-03 00:31:54
[1824] Sinobi2014-03-03 00:31:54

Ezt nem tudom. Leloned, kerlek?

Előzmény: [1823] w, 2014-02-02 22:04:19
[1823] w2014-02-02 22:04:19

Adott az ABCD négyszög, valamint az AC átlón a P és Q; a BD átlón az R és S pontok. A PR, QS, PS, QR egyenesek kilenc négyszögre osztják ABCD-t "3x3-as táblázat alakban".

Tegyük fel, hogy a csúcsokhoz legközelebbi négy kis négyszög érintőnégyszög. Igazoljuk, hogy a középső is az!

[1822] w2014-02-02 21:56:06

Nekem a kérdés szerkesztéselméletinek tűnik. Ha adottak a kúpszelet paraméterei, akkor elvileg megszerkeszthetők azok az együtthatók, melyekre Ax2+Bxy+Cy2+Dx+Ey+F=0 a kúpszelet egyenlete. Feltehetően elmetszettük az x2+y2=1 körrel; behelyettesítéssel és négyzetreemeléssel adódik, hogy a metszéspontok x és y koordinátái egy-egy szerkeszthető együtthatójú, legfeljebb negyedfokú egyenlet gyökei. Ha csak egy metszéspont adott, nem csoda, hogy bizonyos esetben szerkeszthetetlen a többi, hisz ahhoz egy harmadfokú egyenlet gyökeit kellene megszerkeszteni, ami mint tudjuk, nem mindig tehető meg. Ha két vagy három metszéspont adott, akkor a maradék metszéspont koordinátáira egy legfeljebb másodfokú egyenlet adódik, az pedig megoldható szerkesztéssel. Tehát ha 2 vagy 3 metszéspont adott, mindig meg lehet csinálni, ha 0 vagy 1, akkor pedig nem mindig.

Előzmény: [1816] Sinobi, 2014-01-30 09:54:23
[1821] Sinobi2014-02-01 23:36:43

Ket korivnek csak akkor lehet vegtelen sok kozos pontjuk, ha ugyanannak a kornek az ivei. Legyen ez a kor k.

Legyen k sugara r, a kibovitett sinus tetel miatt vagy alfa=beta, vagy alfa=180-beta.

A szelotetel miatt egy szogszar egy k korbol csak akkor metsz ki ket a hosszu szakaszt, ha a szakaszok, es a kor tukrosek a szogszar belso szogfelezojere.

Nem nehez konstrukciot adni, hogy minden alfa=beta<90 -re, es minden alfa=180-beta -ra letezik ilyen szogszar, a hosszu szakaszokkal.

Es alfa=beta>=90 nem jo, mert a belso szogfelezo ket felsikra osztja a sikot, amelyik elvalasztja a latokoriveket, az egyiknek csak az egyik felsikban, a masiknak csak a masik felsikban lesz pontja.

Előzmény: [1820] gergomo, 2014-02-01 22:13:15
[1820] gergomo2014-02-01 22:13:15

Help, valaki! 14 feladatból ebbe belesültünk: Egy szög két szárán adott 1-1 "a" hosszúságú szakasz. Az egyik szakasz alfa szögű látószögkörívének végtelen sok közös pontja van a másik szakasz béta szögű látószögkörívével. Milyen alfa és béta szög esetén lehetséges ez? és miért?:D

[1819] Sinobi2014-01-31 21:42:56

Abbol, hogy 1 db kozos pontjuk van megadva, nem kovetkezik, hogy a tobbi szerkesztheto. Ellenpelda: a Bolyai-féle szögharmadolás.

A 2 db kozos pont esetrol tudsz valami jot?

Előzmény: [1818] Fálesz Mihály, 2014-01-31 14:31:09
[1818] Fálesz Mihály2014-01-31 14:31:09

A Bolyai-féle szögharmadolás:

Előzmény: [1817] jonas, 2014-01-31 10:28:25
[1817] jonas2014-01-31 10:28:25

Ez egy érdekes kérdés, tetszik. Valamiért azt gondoltam elsőre, hogy mindig meg lehet szerkeszteni az összes metszéspontot, de most már nem vagyok biztos benne.

Előzmény: [1816] Sinobi, 2014-01-30 09:54:23
[1816] Sinobi2014-01-30 09:54:23

Ha adott egy kupszelet es egy kor es 0/1/2/3 metszespontjuk, akkor mely esetekben szerkesztheto meg a tobbi metszespont, es mely esetekben nem?

Előzmény: [1786] HoA, 2014-01-05 18:54:23
[1815] Loiscenter2014-01-24 20:25:57

köszi - tanulmányozom a magyarazatot!!!

Előzmény: [1814] w, 2014-01-24 13:42:00
[1814] w2014-01-24 13:42:00

Igen. Még arra sincs szükség, hogy A és B azonos sebességgel mozogjon.

Elég azokat az időpontokat vizsgálni, melyekben az A golyó az eredeti A0 pontban van. Tegyük fel, hogy ameddig A elmegy B0-ba és vissza A0-ba, annyi idő alatt B s.I utat tesz meg, ahol I a félkörív hossza. Célunk az, hogy belássuk, hogy tetszőleges a>0-hoz van olyan n pozitív egész, melyre ns kisebb, mint a távolságra van egy egész számtól.

Ha s racionális szám, akkor nincs mit bizonyítani, létezik olyan n>0 egész, melyre ns egész szám lesz.

Ha s irracionális, akkor pedig tekintsünk egy hatalmas nagy N-et, és az \left[\frac{i}{N};\frac{i+1}N\right) intervallumokat i=0,1,...,N-1-re. Mivel s irracionális, ezért {ns} (ns törtrésze) minden n>0-ra más és más értéket vesz fel. Ezért az {ns} számokat minden n=1,2,...,N+1-re véve, ez az N+1 szám benne van előbbi intervallumok egyikében. Skatulya-elv szerint így lesz két n, n1 és n2, melyre {n1s} és {n2s} ugyanabban az intervallumban van, eltérésük <N-1. Másfelől mivel triviálisan \left\{\{n_1s\}-\{n_2s\}\right\}=\{(n_1-n_2)s\}, ezért kaptunk egy olyan n=n1-n2 számot, melyre {ns}<N-1, vagyis |n|s egy egész számtól kisebb, mint N-1 távolságra van. Márpedig N-1 tetszőlegesen kicsi lehet. (A Kronecker-tételre is hivatkozhattunk volna.)

Összességében tetszőlegesen kis ívhossz lehet A és B távolsága, amiből következik az állítás.

Előzmény: [1813] Loiscenter, 2014-01-23 23:25:25
[1813] Loiscenter2014-01-23 23:25:25
[1812] w2014-01-16 13:06:53

Szép és elemi kérdés, a bizonyítás is nagyon szép, a vélemény szerintem helytálló. A probléma egyébként az IMO 1991/5. feladat.

Persze lehetne vitatkozni, bőven vannak még geometriai szépségek, például ez egészen szép és egyszerű, ám ránézésre egyáltalán nem triviális tétel.

Előzmény: [1811] Loiscenter, 2014-01-16 11:13:32
[1811] Loiscenter2014-01-16 11:13:32

Vietnami matematikusok önkéntes versenyérol( legszebb és legelemibb bizonyitás) idézem egy feladatot :

Az ABC háromszög belsejében van egy P pont. bizonyitando hogy a PAB , PBC, PCA szögek között van 30° -nál nem nagyobb!

kivancsi vagyok a véleményetekre!

[1810] w2014-01-12 23:35:37

Igazad van, nem voltam teljesen alapos, de nem nehéz kipontosítani (látószögek...).

Előzmény: [1809] Loiscenter, 2014-01-12 23:17:35
[1809] Loiscenter2014-01-12 23:17:35

Az állitás hogy ABX vagy ABY közé irható kör sugara nagyobb mint ABC -énél - ezt nem látom!!!

Előzmény: [1808] w, 2014-01-12 21:41:09
[1808] w2014-01-12 21:41:09

Tekintsük a legnagyobb olyan kört, amely a sokszög valamely három csúcsa köré írható, a három csúcs legyen A,B,C. Belátjuk, hogy ez tartalmazza az egész sokszöget, és három szomszédos csúcson halad át.

Ha ez a kör minden csúcsot tartalmaz, akkor egyben tartalmazza az egész sokszöget. Ezért ha nem tartalmazza az egész sokszöget, akkor van olyan X csúcs, amelyet a kör nem tartalmaz. De ekkor ABX kör nagyobb sugarú lesz az ABC körnél, ami ellentmond választásunknak.

Tegyük fel, hogy nincs olyan három szomszédos csúcs, amelyre a köréjük írt kör sugara maximális. Ekkor az előbb igazoltak szerint van, mondjuk körüljárás szerint A és B között, egy olyan Y csúcs, amely a legnagyobb körön belül található. De ekkor az ABY kör sugara ismét nagyobb lesz, mint ABC sugara, ez ellentmondás.

Előzmény: [1807] Loiscenter, 2014-01-12 18:55:23
[1807] Loiscenter2014-01-12 18:55:23

Egy konvex sokszögnél mindig megtalalható 3 egymast követö csucsot, melyekre a köré irható kör tartalmazza az egész sokszöget?( elemi egyszerü bizonyitas jó lenne)

[1806] epsilon2014-01-12 16:29:01

Köszi a linket Kemény Legény! Igen, az összegnél elírtam, kösz a javítást, de utólag láttam, a sejtésem úgy sem talál Üdv: epsilon

Előzmény: [1804] Kemény Legény, 2014-01-12 11:04:39
[1805] fityfiritty2014-01-12 13:04:51

És egy apróság: \frac{n (n + 1)}2 \ne \frac{(n - 2)(n - 1)}2 .

Előzmény: [1803] epsilon, 2014-01-11 16:13:13
[1804] Kemény Legény2014-01-12 11:04:39

Az alábbi linken van pár bizonyítás a háromszöges esetre: itt.

Előzmény: [1803] epsilon, 2014-01-11 16:13:13
[1803] epsilon2014-01-11 16:13:13

Ha nem tévedek, a kérdésre a válasz az alábbi lenne, de nem tudom bizonyítani, sem a tetraéderre átültetni :-(

Előzmény: [1802] epsilon, 2014-01-11 15:17:36
[1802] epsilon2014-01-11 15:17:36

Üdv mindenkinek! A felvetett hasáb számlálási feladat kapcsán előkerült egy másik feladat is: Egy szabályos háromszög minden oldalát osszuk fel n egyenlő részre, majd ezeken át húzzunk az oldalakkal párhuzamos egyeneseket. Így a háromszöget n×n kis kongruens háromszögre osztottuk. Összesen hány háromszög látható az ábrán? Sajnos az eredményre zárt alakban nincs tippem. Olyan megoldás érdekelne, amelyik kiterjeszthető a térbe is, ha a szabályos háromszög helyett szabályos tetraédert tekintünk, és azt bontjuk kis tetraéderekre. Tudna-e valaki segíteni? Előre is köszönöm, üdv: epsilon

Előzmény: [1796] Kemény Legény, 2014-01-08 10:23:09
[1801] HoA2014-01-09 12:38:28

Rajzoljunk a két szögszár teljes egyenesére egy-egy „centiméter” skálát. Azokat a pontpárokat összekötve, melyek távolságának összege a metszésponttól egy adott ( előjeles ) érték, Steiner II. tételének duálisa szerint szintén egy másodrendű görbe érintőit kapjuk, melyek között a végtelen távoli egyenes (mint a megfelelő pontokat összekötő egyenes) is ott van. Ekkor a burkolt görbe parabola.

A feladat feltételeinek megfelelő, a beírt kör O középpontján áthaladó, a szögfelezőre szimmetrikus egyenespárt tehát akár [1799] szerint O-ból húzott hiperbola érintőként, akár az itteni parabola érintőiként megrajzolhatjuk. Az [1800] –ban felvetett egzisztencia kérdés persze vizsgálandó: Kívül esik-e O az érintendő görbéken?

Előzmény: [1800] marcius8, 2014-01-09 12:30:07

  [1]    [2]    [3]    [4]    [5]    [6]    [7]    [8]    [9]    [10]    [11]    [12]    [13]    [14]    [15]    [16]    [17]    [18]    [19]    [20]    [21]    [22]    [23]    [24]    [25]    [26]    [27]    [28]    [29]    [30]    [31]    [32]    [33]    [34]    [35]    [36]    [37]    [38]    [39]    [40]    [41]    [42]    [43]    [44]    [45]    [46]    [47]    [48]    [49]    [50]    [51]    [52]    [53]    [54]    [55]    [56]    [57]    [58]    [59]    [60]    [61]    [62]    [63]    [64]    [65]    [66]    [67]    [68]    [69]    [70]    [71]    [72]    [73]    [74]    [75]    [76]    [77]    [78]